Toán học - Bất đẳng thức, các bài toán chọn lọc

pdf88 trang | Chia sẻ: minhhong95 | Lượt xem: 946 | Lượt tải: 1download
Bạn đang xem trước 20 trang mẫu tài liệu Toán học - Bất đẳng thức, các bài toán chọn lọc, để tải tài liệu gốc về máy bạn click vào nút DOWNLOAD ở trên
Võ Quốc Bá Cẩn
Bất đẳng thức
Các bài toán chọn lọc
Ngày 19 tháng 4 năm 2007
ii
Mục lục
1 Các bài toán đã có lời giải 1
iii
iv MỤC LỤC
Chương 1
Các bài toán đã có lời giải
1.1 [Võ Quốc Bá Cẩn] Cho x, y, z là các số dương thỏa xy + yz + zx = 1, chứng minh
1√
1 + (2x− y)2 +
1√
1 + (2y − z)2 +
1√
1 + (2z − x)2 ≤
3
√
3
2
Lời giải. Đặt a = 2x− y, b = 2y− z, c = 2z− x, do đó a+ b+ c = x+ y+ z > 0 và từ xy+ yz+ zx = 1,
ta có
14(a2 + b2 + c2) + 35(ab+ bc+ ca) = 49
Lại có 3(14(a2 + b2 + c2) + 35(ab+ bc+ ca)) ≤ 49(a+ b+ c)2, nên
a+ b+ c ≥
√
3
Ta sẽ chứng minh với mọi số thực a, b, c thỏa mãn a+ b+ c ≥ √3, thì
P (a, b, c) =
1√
a2 + 1
+
1√
b2 + 1
+
1√
c2 + 1
≤ 3
√
3
2
Nếu c ≤ 0, thay c bởi c′ = −c, thì ta cũng có a+b+c′ ≥ √3, và giá trị của biểu thức P vẫn không đổi,
do đó, không mất tính tổng quát, ta có thể giả sử a, b, c > 0, khi đó, đặt a = ka1, b = kb1, c = kc1
với k ≥ 1, a1, b1, c1 > 0 sao cho a1 + b1 + c1 =
√
3, thì
P (a, b, c) =
∑
cyc
1√
k2a21 + 1
≤
∑
cyc
1√
a21 + 1
= P (a1, b1, c1)
Như vậy, ta có thể giả sử a, b, c > 0 và a+ b+ c =
√
3. Xét hàm số f(x) = 1√
x2+1
, ta có
f ′′(x) =
2x2 − 1
(x2 + 1)5/2
Từ đây, ta có thể dễ dàng kiểm tra được f lõm trên
[
0, 1√
2
]
và lồi trên
[
1√
2
,
√
3
]
.
Không mất tính tổng quát, giả sử a ≥ b ≥ c > 0, từ đây suy ra c ≤ 1√
3
, Xét 2 trường hợp
Trường hợp 1. b ≤ 1√
2
, sử dụng bất đẳng thức Jensen
f(b) + f(c) ≤ 2f
(
b+ c
2
)
=
2√(
b+c
2
)2
+ 1
=
4√(√
3− a)2 + 4
Ta cần chứng minh
4√(√
3− a)2 + 4 +
1√
a2 + 1
≤ 3
√
3
2
(1.1)
1
2 CHƯƠNG 1. CÁC BÀI TOÁN ĐÃ CÓ LỜI GIẢI
Thật vậy, đặt a = t√
3
thì 3 ≥ t ≥ 1 và ta cần chứng minh
4√
t2 − 6t+ 21 +
1√
t2 + 3
≤ 3
2
Hay
16
t2 − 6t+ 21 +
1
t2 + 3
+
8
√
(t2 + 3)(t2 − 6t+ 21)
(t2 + 3)(t2 − 6t+ 21) ≤
9
4
Sử dụng bất đẳng thức AM - GM, ta có√
t2 + 3 ≤ t
2 + 7
4
,
√
t2 − 6t+ 21 ≤ t
2 − 6t+ 37
8
Như vậy, ta chỉ cần chứng minh
16
t2 − 6t+ 21 +
1
t2 + 3
+
(t2 + 7)(t2 − 6t+ 37)
4(t2 + 3)(t2 − 6t+ 21) ≤
9
4
Hay
(t− 1)2(t− 2)2 ≥ 0
Bất đẳng thức này hiển nhiên đúng.
Trường hợp 2. b ≥ 1√
2
, ta có
f(a) + f(b) ≤ f
(
1√
2
)
+ f
(
a+ b− 1√
2
)
Sử dụng bất đẳng thức Jensen,
f
(
1√
2
)
+ f(c) ≤ 2f
(
c+ 1√
2
2
)
= 2f
√3−
(
a+ b− 1√
2
)
2

Như vậy, ta cần chứng minh
2f
√3−
(
a+ b− 1√
2
)
2
+ f (a+ b− 1√
2
)
≤ 3
√
3
2
Bất đẳng thức này đúng theo (1.1). Bất đẳng thức được chứng minh xong.
Đẳng thức xảy ra khi và chỉ khi x = y = z = 1√
3
.
Nhận xét. Bất đẳng thức trên vẫn đúng với mọi x, y, z ∈ R thỏa mãn xy + yz + zx = 1.
1.2 Cho các số dương a, b, c thỏa abc = 1, chứng minh rằng
a
√
b+ c
b+ c+ 1
+
b
√
c+ a
c+ a+ 1
+
c
√
a+ b
a+ b+ 1
≥
√
2
Lời giải. Sử dụng bất đẳng thức Holder, ta có(∑
cyc
a
√
b+ c
b+ c+ 1
)2(∑
cyc
a(b+ c+ 1)2
b+ c
)
≥ (a+ b+ c)3
Do đó, ta cần chứng minh
(a+ b+ c)3 ≥ 2
∑
cyc
a(b+ c+ 1)2
b+ c
3hay ∑
cyc
a3 + 3
∑
cyc
a
b
+ 3
∑
cyc
b
a
+ 6 ≥ 4
∑
cyc
ab+ 4
∑
cyc
a+ 2
∑
cyc
a
b+ c
Sử dụng bất đẳng thức AM - GM, ta lại có∑
cyc
a
b
≥
∑
cyc
ab,
∑
cyc
b
a
≥
∑
cyc
ab, 2
∑
cyc
a
b+ c
≤ 1
2
∑
cyc
a
b
+
1
2
∑
cyc
b
a
Do đó,
V T − V P ≥
∑
cyc
a3 +
5
2
∑
cyc
a
b
+
5
2
∑
cyc
b
a
− 4
∑
cyc
ab− 4
∑
cyc
a+ 6
≥
∑
cyc
a3 +
∑
cyc
ab− 4
∑
cyc
a+ 6 =
∑
cyc
(
a3 − 4a+ 1
a
+ 2
)
Xét hàm số f(x) = x3 − 4x+ 1x + 2 + 2 lnx với x > 0, ta có
f ′(x) = (x− 1)
(
3x+ 3 +
1
x2
− 1
x
)
Nếu x ≤ 1 thì 1x2 ≥ 1x , nếu x ≥ 1 thì 1 ≥ 1x , do đó
f ′(x) = 0⇔ x = 1
Từ đây, ta dễ dàng kiểm tra được
f(x) ≥ f(1) = 0 ∀x > 0
Hay
x3 − 4x+ 1
x
+ 2 ≥ −2 lnx ∀x > 0
Vậy ∑
cyc
(
a3 − 4a+ 1
a
+ 2
)
≥ −2
∑
cyc
ln a = 0
Bất đẳng thức được chứng minh xong.
Đẳng thức xảy ra khi và chỉ khi a = b = c = 1.
1.3 [Phạm Kim Hùng] Với mọi số không âm a, b, c, ta có√
a
4a+ 4b+ c
+
√
b
4b+ 4c+ a
+
√
c
4c+ 4a+ b
≤ 1
Lời giải. Sử dụng bất đẳng thức Cauchy Schwarz, ta có∑
cyc
√
a
4a+ 4b+ c
≤
√
3
∑
cyc
a
4a+ 4b+ c
Không mất tính tổng quát, giả sử a+ b+ c = 3 và b là số hạng nằm giữa a và c, ta cần chứng minh∑
cyc
a
a+ b+ 1
≤ 1
hay
a2b+ b2c+ c2a+ abc ≤ 4
4 CHƯƠNG 1. CÁC BÀI TOÁN ĐÃ CÓ LỜI GIẢI
Vì b là số hạng nằm giữa a và c nên
c(b− a)(b− c) ≤ 0
Suy ra
b2c+ c2a ≤ abc+ bc2
Do đó
a2b+ b2c+ c2a+ abc ≤ b(a+ c)2 ≤ 1
2
(
2b+ (a+ c) + (a+ c)
3
)3
= 4
Bất đẳng thức được chứng minh xong.
Đẳng thức xảy ra khi và chỉ khi a = b = c.
1.4 [Phạm Hữu Đức] Cho các số dương a, b, c, chứng minh
1
a2 + bc
+
1
b2 + ca
+
1
c2 + ab
≤ a+ b+ c
ab+ bc+ ca
(
1
a+ b
+
1
b+ c
+
1
c+ a
)
Lời giải. Ta có bất đẳng thức cần chứng minh tương đương với∑
cyc
z(a2 − b2)(a− b) ≥ 0
Với
x = (a(b+ c)(b2 + c2) + 2a2(b2 + c2) + 3a2bc+ a3(b+ c)− b2c2)(a2 + bc)
y = (b(c+ a)(c2 + a2) + 2b2(c2 + a2) + 2b2ca+ b3(c+ a)− c2a2)(b2 + ca)
z = (c(a+ b)(a2 + b2) + 2c2(a2 + b2) + 2c2ab+ c3(a+ b)− a2b2)(c2 + ab)
Không mất tính tổng quát, giả sử a ≥ b ≥ c > 0, khi đó dễ thấy x, y ≥ 0. Lại có
y + z ≥ b(c+ a)(c2 + a2)(b2 + ca)− a2b2(c2 + ab)
≥ a3b(b2 + ca)− a2b2(c2 + ab) = a2bc(a2 − bc) ≥ 0
Chú ý rằng a ≥ b ≥ c > 0 nên (c2 − a2)(c− a) ≥ (a2 − b2)(a− b). Từ đây, ta có đpcm.
Đẳng thức xảy ra khi và chỉ khi a = b = c hoặc a = t > 0, b = c→ 0 và các hoán vị.
Nhận xét. Bài toán này có thể giải dễ dàng nhờ phép đặt p, q, r.
1.5 Chứng minh rằng với mọi số dương a, b, c ta luôn có
a3
2a2 − ab+ 2b2 +
b3
2b2 − bc+ 2c2 +
c3
2c2 − ca+ 2a2 ≥
a+ b+ c
3
Lời giải. Ta có bất đẳng thức cần chứng minh tương đương với∑
cyc
Sc(a− b)2 ≥ 0
trong đó
Sa =
2c− b
2b2 − bc+ 2c2 , Sb =
2a− c
2c2 − ca+ 2a2 , Sc =
2b− a
2a2 − ab+ 2b2
Xét 2 trường hợp
Trường hợp 1. a ≥ b ≥ c > 0, khi đó, dễ thấy Sb ≥ 0. Ta sẽ chứng minh
Sb + 2Sc ≥ 0 (1)
5a2Sb + 2b2Sa ≥ 0 (2)
Thật vậy, ta có
(1)⇔ 6a2b+ 4ab2 − 3abc+ 8bc2 − 4ac2 − 2b2c ≥ 0 (đúng do a ≥ b ≥ c)
(2)⇔ f(a) = a
2(2a− c)
2c2 − ca+ 2a2 +
2b2(2c− b)
2b2 − bc+ 2c2 ≥ 0
Lại có
f ′(a) =
a(4a3 − 4a2c+ 13ac2 − 4c3)
(2c2 − ca+ 2a2)2 ≥ 0
Do đó, f(a) là hàm đồng biến. Suy ra,
f(a) ≥ f(b) = 3b
2c
2b2 − bc+ 2c2 ≥ 0
Các bất đẳng thức (1) và (2) được chứng minh.
Từ các bất đẳng thức này và với chú ý rằng a ≥ b ≥ c > 0 nên (a−c)2 ≥ max
{
a2
b2 ·(b− c)2, (a− b)2
}
,
ta có
2
∑
cyc
Sc(a− b)2 =
(
Sb(c− a)2 + 2Sa(b− c)2
)
+
(
Sb(c− a)2 + 2Sc(a− b)2
)
≥ (b− c)
2
b2
(a2Sb + 2b2Sa) + (a− b)2(Sb + 2Sc) ≥ 0
Trường hợp 2. c ≥ b ≥ a > 0, dễ thấy Sc, Sa ≥ 0. Nếu 2a ≥ c thì bất đẳng thức cần chứng minh
hiển nhiên đúng. Xét trường hợp ngược lại c ≥ 2a, tức là Sb ≤ 0. Xét 2 trường hợp nhỏ
Trường hợp 2.1. 2b ≥ c+ a, ta có
Sb(c− a)2 + Sc(a− b)2 ≥ 0 (3)
⇔ m(b) = (a− b)
2(2b− a)
2a2 − ab+ 2b2 +
(c− a)2(2a− c)
2c2 − ca+ 2a2 ≥ 0
m′(b) =
(b− a)(4b3 + 9a2b− 7a3)
(2a2 − ab+ 2b2)2 ≥ 0
Do đó, m(b) là hàm đồng biến. Suy ra,
m(b) ≥ m
(
a+ c
2
)
=
a(a− c)2(16a2 − 2ac+ c2)
2(4a2 + ac+ c2)(2a2 − ac+ 2c2) ≥ 0
Vậy (3) đúng. Do đó, ∑
cyc
Sc(a− b)2 ≥ Sb(c− a)2 + Sc(a− b)2 ≥ 0
Trường hợp 2.2. c+ a ≥ 2b.
Trường hợp 2.2.1. 2b− a ≥ 4a, ta sẽ chứng minh
Sc + 3Sb ≥ 0 (4)
Sa +
3
2
Sb ≥ 0 (5)
Thật vậy
(4)⇔ g(c) = 2b− a
2a2 − ab+ 2b2 +
3(2a− c)
2c2 − ca+ 2a2 ≥ 0
6 CHƯƠNG 1. CÁC BÀI TOÁN ĐÃ CÓ LỜI GIẢI
Ta có
g′(c) =
6c(c− 4a)
(2c2 − ca+ 2a2)2 ≥ 0
Do đó, g(c) là hàm đồng biến. Suy ra,
g(c) ≥ g(2b− a) = 4b
3 − 4ab2 − a2b+ 13a3
(2a2 − ab+ 2b2)(5a2 − 10ab+ 8b2) ≥ 0
(5)⇔ h(a) = 4c− 2b
2b2 − bc+ 2c2 +
6a− 3c
2c2 − ca+ 2a2 ≥ 0
h′(a) =
3(3c2 + 4ca− 4a2)
(2c2 − ca+ 2a2)2 ≥ 0
Do đó, h(a) là hàm đồng biến. Suy ra,
++, Nếu c ≥ 2b thì
h(a) ≥ h(0) = (c− 2b)(2c+ 3b)
2c(2c2 − bc+ 2b2) ≥ 0
++, Nếu 2b ≥ c thì
h(a) ≥ h(2b− c) = (2b− c)(4b
2 + 13bc− 2c2)
(2b2 − bc+ 2c2)(8b2 − 10bc+ 5c2) ≥ 0
Tóm lại, ta luôn có h(a) ≥ 0.
Từ (4) và (5) với chú ý rằng (c− a)2 ≤ 3(b− a)2 + 32 (b− c)2, ta có∑
cyc
Sc(a− b)2 ≥ (Sc + 3Sb)(a− b)2 +
(
Sa +
3
2
Sb
)
(b− c)2 ≥ 0
Trường hợp 2.2.2. 2b− a ≤ 4a⇔ a ≥ 25b, ta có
Sa + Sb + Sc ≥ 0 (6)
SaSb + SbSc + ScSa ≥ 0 (7)
(6) hiển nhiên đúng vì theo (5), ta có
Sa + Sb + Sc = Sa +
(
3
2
Sb + Sc
)
− 1
2
Sb ≥ 0
Bây giờ ta sẽ chứng minh (7), ta có
(7)⇔ k(c) = 4(ab3 + bc3 + ca3) + 7abc(a+ b+ c)− 2(a3b+ b3c+ c3a)
− 6(a2b2 + b2c2 + c2a2) ≥ 0
k′(c) = 12bc2 + 4a3 + 14abc+ 7ab(a+ b)− 2b3 − 6ac2 − 12c(a2 + b2)
k′′(c) = 24bc− 12ac+ 14ab− 12a2 − 12b2
≥ 24b2 − 12ab+ 14ab− 12a2 − 12b2 = 12b2 + 2ab− 12a2 ≥ 0
Do đó, k′(c) là hàm đồng biến. Suy ra,
k′(c) ≥ k′(b) = 4a3 − 5a2b+ 15ab2 − 2b3 ≥ 0 (do a ≥ 2
5
b)
Suy ra, k(c) là hàm đồng biến. Do đó,
k(c) ≥ k(b) = b(2a3 − 5a2b+ 16ab2 − 4b3) ≥ 0 (do a ≥ 2
5
b)
Từ đây, ta có đpcm.
Đẳng thức xảy ra khi và chỉ khi a = b = c.
71.6 [Võ Quốc Bá Cẩn] Cho các số không âm a, b, c thỏa a+ b+ c = 1. Chứng minh bất đẳng thức√
a+
(b− c)2
4
+
√
b+
(c− a)2
4
+
√
c+
(a− b)2
4
≤
√
3 +
(
1−
√
3
2
)
(|a− b|+ |b− c|+ |c− a|)
Lời giải. Không mất tính tổng quát, giả sử a ≥ b ≥ c ≥ 0. Đặt a+ b = 2t, a− b = 2m, k = 14 thì do giả
thiết, ta có t ≥ m ≥ t− c ≥ 0. Khi đó, bất đẳng thức cần chứng minh trở thành
f(m) =
√
t+m+ k(m+ c− t)2 +
√
t−m+ k(m+ t− c)2 +
√
c+ 4km2
−
(
2−
√
3
)
(t+m− c) ≤
√
3
Ta có
f ′′(m) =
4k(2t− c)− 1
4 (t+m+ k(m+ c− t)2)3/2
+
4k(2t− c)− 1
4 (t−m+ k(m+ t− c)2)3/2
+
4kc
(c+ 4km2)3/2
= − c
2
(
a+ 14 (b− c)2
)3/2 − c2 (b+ 14 (c− a)2)3/2 +
c(
c+ 14 (a− b)2
)3/2
Chú ý rằng a ≥ b ≥ c ≥ 0 nên
a+
1
4
(b− c)2 − c− 1
4
(a− b)2 = 3
4
(a− c)(b+ 1) ≥ 0
b+
1
4
(c− a)2 − c− 1
4
(a− b)2 = 3
4
(b− c)(a+ 1) ≥ 0
Suy ra (
a+
1
4
(b− c)2
)3/2
≥
(
c+
1
4
(a− b)2
)3/2
≥ 0(
b+
1
4
(c− a)2
)3/2
≥
(
c+
1
4
(a− b)2
)3/2
≥ 0
Do đó,
f ′′(m) ≥ − c
2
(
c+ 14 (a− b)2
)3/2 − c2 (c+ 14 (a− b)2)3/2 +
c(
c+ 14 (a− b)2
)3/2 = 0
Suy ra, f(m) là hàm lồi. Do đó,
f(m) ≤ max {f(0), f(t− c)}
Như vậy, ta chỉ cần chứng minh
max {f(0), f(t− c)} ≤
√
3
Điều này có nghĩa là ta chỉ cần chứng minh bất đẳng thức đã cho trong trường hợp 3 số a, b, c có
2 số bằng nhau, không mất tính tổng quát, giả sử b = c. Ta cần chứng minh
√
a+ 2
√
b+
(a− b)2
4
≤
√
3 +
(
2−
√
3
)
|a− b|
Hay √(
3a− 1
2
)2
+ 2(1− a) ≤
√
3 +
(
1−
√
3
2
)
|3a− 1| − √a
8 CHƯƠNG 1. CÁC BÀI TOÁN ĐÃ CÓ LỜI GIẢI
Đặt t =
√
3a thì ta có t ≤ √3, ta cần chứng minh√
3t4 − 14t2 + 27 ≤ 6− 2t+
(
2
√
3− 2
)
|t2 − 1|
Xét 2 trường hợp
Trường hợp 1. t ≥ 1, ta có bất đẳng thức tương đương√
3t4 − 14t2 + 27 ≤ 6− 2t+
(
2
√
3− 2
)
(t2 − 1)
Hay
2(t− 1)
(
t−
√
3
)((
6
√
3− 9
)
t2 +
(
3 +
√
3
)
t+ 18− 11
√
3
)
≤ 0
Bất đẳng thức này hiển nhiên đúng do
√
3 ≥ t ≥ 1.
Trường hợp 2. t ≤ 1, bất đẳng thức trở thành√
3t4 − 14t2 + 27 ≤ 6− 2t−
(
2
√
3− 2
)
(t2 − 1)
Hay
2(t− 1)
((
6
√
3− 9
)
t3 +
(
2
√
3− 3
)
t2 +
(
2
√
3− 9
)
t+ 6
√
3− 3
)
≤ 0
Bất đẳng thức này cũng đúng do 1 ≥ t ≥ 0. Bài toán được giải quyết hoàn toàn.
Đẳng thức xảy ra khi và chỉ khi a = b = c = 13 hoặc a = 1, b = c = 0 và các hoán vị.
Nhận xét. Ta có 1 kết quả "yếu" hơn nhưng khá "đẹp" là√
a+
(b− c)2
4
+
√
b+
(c− a)2
4
+
√
c+
(a− b)2
4
≤ 2
với mọi a, b, c ≥ 0, a+ b+ c = 1.
1.7 [Phạm Kim Hùng] Cho các số dương a, b, c thỏa a+ b+ c = 3, chứng minh bất đẳng thức
a3/2b+ b3/2c+ c3/2a ≤ 3
Lời giải. Sử dụng bất đẳng thức Cauchy Schwarz, ta có
a3/2b+ b3/2c+ c3/2a ≤
√
(ab+ bc+ ca)(a2b+ b2c+ c2a)
Như vậy, ta chỉ cần chứng minh
(ab+ bc+ ca)(a2b+ b2c+ c2a) ≤ 9
Hay
(ab+ bc+ ca)(a+ b+ c)(a2b+ b2c+ c2a) ≤ 27
Hay
(ab+ bc+ ca)
(∑
cyc
a3b+
∑
cyc
a2b2 + 3abc
)
≤ 27
Chú ý rằng 12
∑
cyc(a
2 − c2 − 2ab+ bc+ ca)2 ≥ 0 nên
∑
cyc
a3b ≤ 1
3
(∑
cyc
a2
)2
Ta cần chứng minh
(ab+ bc+ ca)
(∑
cyc
a2
)2
+ 3
∑
cyc
a2b2 + 9abc
 ≤ 81
9Đặt x = ab + bc + ca thì theo bất đẳng thức AM - GM và Schur, ta có x ≤ 3, 3abc ≥ 4x − 9, bất
đẳng thức trở thành
x((9− 2x)2 + 3x2 − 9abc) ≤ 81
Như vậy, ta chỉ cần chứng minh
x((9− 2x)2 + 3x2 − 3(4x− 9)) ≤ 81
Hay
(x− 3)(7x2 − 27x+ 27) ≤ 0
Bất đẳng thức này hiển nhiên đúng do 3 ≥ x ≥ 0.
Bất đẳng thức được chứng minh xong.
Đẳng thức xảy ra khi và chỉ khi a = b = c = 1.
1.8 [Phạm Kim Hùng] Chứng minh rằng với mọi số thực a, b, c, ta có
ab
4a2 + b2 + 4c2
+
bc
4b2 + c2 + 4a2
+
ca
4c2 + a2 + 4b2
≤ 1
3
Lời giải. Dễ thấy trong 3 số a, b, c luôn tồn tại ít nhất 2 số cùng dấu, giả sử bc ≥ 0, nếu ab ≤ 0, ac ≤ 0
thì ∑
cyc
ab
4a2 + b2 + 4c2
≤ bc
4b2 + c2 + 4a2
≤ 1
4
<
1
3
Như vậy, ta chỉ cần xét trường hợp các số a, b, c cùng dấu, và do đó, ta chỉ cần xét a, b, c ≥ 0 là đủ.
Không mất tính tổng quát, ta có thể giả sử a2 + b2 + c2 = 3 và b là số hạng nằm giữa a và c, bất
đẳng thức trở thành
4
∑
cyc
ab+ 4
∑
cyc
ab3 + abc
∑
cyc
ab2 + a2b2c2 ≤ 16 + 4
∑
cyc
a2b2
Vì b là số hạng nằm giữa a và c nên
a(b− a)(b− c) ≤ 0
Suy ra
ab2 + a2c ≤ abc+ a2b
Do đó ∑
cyc
ab2 ≤ b(a2 + c2) + abc = 2− (b− 1)2(b+ 2) + abc ≤ 2 + abc
Sử dụng kết quả bài toán trước, ta có
∑
cyc
ab3 ≤ 1
3
(∑
cyc
a2
)2
= 3
Như vậy, ta chỉ cần chứng minh
4
∑
cyc
ab+ abc(2 + abc) + a2b2c2 ≤ 4 + 4
∑
cyc
a2b2
Mặt khác, sử dụng bất đẳng thức AM - GM, ta có
2abc ≤ a2b2c2 + 1
Do đó, ta phải chứng minh
P (a, b, c) = 4
∑
cyc
a2b2 − 4
∑
cyc
ab− 3a2b2c2 + 3 ≥ 0
10 CHƯƠNG 1. CÁC BÀI TOÁN ĐÃ CÓ LỜI GIẢI
Vì đây là một bất đẳng thức đối xứng với a, b, c nên không mất tính tổng quát, giả sử a = max{a, b, c}
suy ra a ≥ 1, b2 + c2 ≤ 2, ta có
P (a, b, c)− P
(
a,
√
b2 + c2
2
,
√
b2 + c2
2
)
= (b− c)2
(
2 +
4a√
2(b2 + c2) + b+ c
+
(3a2 − 4)(b+ c)2
4
)
Ta có
2 +
(3a2 − 4)(b+ c)2
4
≥ 2− (b+ c)
2
4
≥ 2− b
2 + c2
2
> 0
Do đó
P (a, b, c) ≥ P
(
a,
√
b2 + c2
2
,
√
b2 + c2
2
)
Như vậy, ta chỉ cần chứng minh
P (a, t, t) ≥ 0
với a ≥ t ≥ 0, a2 + 2t2 = 3.
Hay
(a2 + 2t2)3 + 12t2(a2 + 2t2)(2a2 + t2) ≥ 27a2t4 + 4t(2a+ t)(a2 + 2t2)2
Hay
(a− t)2(a2(a− 3t)2 + 4a2t2 + 16t4) ≥ 0
Bất đẳng thức này hiển nhiên đúng. Vậy ta có đpcm.
Đẳng thức xảy ra khi và chỉ khi a = b = c.
1.9 [Phan Hồng Sơn] Cho các số không âm a, b, c thỏa a+ b+ c = 3, chứng minh√
a2 + b2
(a+ 1)(b+ 1)
+
√
b2 + c2
(b+ 1)(c+ 1)
+
√
c2 + a2
(c+ 1)(a+ 1)
≥ 3√
2
Lời giải. Bình phương 2 vế rồi nhân 2 vế với (a+ 1)(b+ 1)(c+ 1), ta được bất đẳng thức tương đương∑
cyc
(a2 + b2)(1 + c) + 2
∑
cyc
√
(a2 + c2)(b2 + c2)(a+ 1)(b+ 1) ≥ 9
2
(a+ 1)(b+ 1)(c+ 1)
Theo bất đẳng thức Cauchy Schwarz thì∑
cyc
√
(a2 + c2)(b2 + c2)(a+ 1)(b+ 1) ≥
∑
cyc
(c2 + ab)
(
1 +
√
ab
)
Ta cần chứng minh∑
cyc
(a2 + b2)(1 + c) + 2
∑
cyc
(c2 + ab)
(
1 +
√
ab
)
≥ 9
2
(a+ 1)(b+ 1)(c+ 1)
Hay
8
∑
cyc
a2 +
∑
cyc
ab+ 4
∑
cyc
√
ab(c2 + ab) ≥ 36 + 15abc
Sử dụng bất đẳng thức AM - GM và Schur, ta có
4
∑
cyc
√
ab(c2 + ab)− 15abc ≥ 9abc ≥ 12
∑
cyc
ab− 27
11
Như vậy, ta chỉ cần chứng minh
8
∑
cyc
a2 +
∑
cyc
ab+ 12
∑
cyc
ab− 27 ≥ 36
Hay
ab+ bc+ ca ≤ 3
Bất đẳng thức này đúng theo bất đẳng thức AM - GM. Vậy ta có đpcm.
Đẳng thức xảy ra khi và chỉ khi a = b = c = 1.
1.10 [Phan Hồng Sơn] Với mọi a ≥ b ≥ c ≥ 0, đặt
P =
a
b+ c
+
b
c+ a
+
c
a+ b
Q =
2(b+ c)− a
4a+ b+ c
+
2(c+ a)− b
4b+ c+ a
+
2(a+ b)− c
4c+ a+ b
Chứng minh rằng
1. Nếu a+ c ≥ 2b thì P ≥ Q.
2. Nếu a+ c ≤ 2b thì P ≤ Q.
Lời giải. Không mất tính tổng quát, giả sử a+ b+ c = 1.
(1) Bất đẳng thức cần chứng minh tương đương với∑
cyc
3a− 1
(3a+ 1)(1− a) ≥ 0
Hay ∑
cyc
z(a− b)2 ≥ 0
với x = (1− 9a2)(1− a), y = (1− 9b2)(1− b), z = (1− 9c2)(1− c).
Chú ý rằng a ≥ b ≥ c, a+ c ≥ 2b nên
b ≤ 1
3
, y, z ≥ 0, a− c ≥ 2(b− c) ≥ 0, a− b ≥ b− c ≥ 0
Do đó, ta chỉ cần chứng minh
x+ 4y + z ≥ 0
Hay
F (a, b, c) = 9(a3 + c3)− 9(a2 + c2) + 36b3 − 36b2 − 3b+ 5 ≥ 0
Ta có
F (a, b, c) =
(1− 3b)(11 + 30b− 45b2 + 9(a− c)2)
4
≥ 0
(2) Bằng biến đổi tương tự, ta có bất đẳng thức tương đương∑
cyc
Sc(a− b)2 ≥ 0
với Sa = (9a2 − 1)(1− a), Sb = (9b2 − 1)(1− b), Sc = (9c2 − 1)(1− c).
Do a ≥ b ≥ c, 2b ≥ a+ c nên
1
2
≥ b ≥ 1
3
, Sa, Sb ≥ 0, a− c ≥ 2(a− b) ≥ 0, b− c ≥ a− b ≥ 0
12 CHƯƠNG 1. CÁC BÀI TOÁN ĐÃ CÓ LỜI GIẢI
Như vậy, ta chỉ cần chứng minh
Sa + 4Sb + Sc ≥ 0
Hay
G(a, b, c) = −9(a3 + c3) + 9(a2 + c2)− 36b3 + 36b2 + 3b− 5 ≥ 0
Ta có
G(a, b, c) =
(3b− 1)(11 + 30b− 45b2 + 9(a− c)2)
4
≥ 0
Bài toán được giải quyết hoàn toàn.
Đẳng thức ở cả 2 bất đẳng thức xảy ra khi và chỉ khi 2b = a+ c.
1.11 [Lê Văn Chánh] Cho các số không âm a, b, c thỏa a+ b+ c = 1, đặt x = a2+ b2+ c2, chứng minh
bất đẳng thức √
1 + 2a2 − x+
√
1 + 2b2 − x+
√
1 + 2c2 − x ≥ √11− 9x
Lời giải. Bình phương 2 vế rồi thu gọn, ta có thể viết lại bất đẳng thức như sau∑
cyc
√
(1 + a2 − b2 − c2)(1 + b2 − c2 − a2) ≥ 8
∑
cyc
ab
Sử dụng bất đẳng thức GM - HM, ta có√
(1 + a2 − b2 − c2)(1 + b2 − c2 − a2) ≥ (1 + a
2 − b2 − c2)(1 + b2 − c2 − a2)
1− c2
Ta cần chứng minh ∑
cyc
(1 + a2 − b2 − c2)(1 + b2 − c2 − a2)
1− c2 ≥ 8
∑
cyc
ab
Hay
2
∑
cyc
c(a− b)2
1 + c
≥ 0
Bất đẳng thức này hiển nhiên đúng, vậy ta có đpcm. Đẳng thức xảy ra khi và chỉ khi a = b = c = 13
hoặc a = 1, b = c = 0 và các hoán vị.
1.12 Chứng minh rằng với mọi a, b, c > 0, ta có
1
a(a+ b)
+
1
b(b+ c)
+
1
c(c+ a)
≥ 3
2(abc)2/3
Lời giải. Không mất tính tổng quát, ta có thể giả sử abc = 1. Khi đó, tồn tại các số dương x, y, z sao
cho a = xy , b =
z
x , c =
y
z , bất đẳng thức trở thành∑
cyc
y2
x2 + yz
≥ 3
2
Sử dụng bất đẳng thức Cauchy Schwarz, ta có∑
cyc
y2
x2 + yz
≥ (x
2 + y2 + z2)2
x2y2 + y2z2 + z2x2 + x3y + y3z + z3x
Mặt khác, ta lại có
(x2 + y2 + z2)2 − 3(x3y + y3z + z3x) = 1
2
∑
cyc
(x2 − z2 − 2xy + yz + zx)2 ≥ 0
(x2 + y2 + z2)2 − 3(x2y2 + y2z2 + z2x2) = 1
2
∑
cyc
(x2 − y2)2 ≥ 0
Nên từ đây, ta dễ dàng suy ra đpcm. Đẳng thức xảy ra khi và chỉ khi a = b = c.
13
1.13 [Phan Thành Nam, VMEO 2005] Chứng minh rằng nếu a, b, c > 0 thì
1
a
√
a+ b
+
1
b
√
b+ c
+
1
c
√
c+ a
≥ 3√
2abc
Lời giải. Tương tự bài trên, ta cũng đưa bài toán về chứng minh rằng với mọi x, y, z > 0 thì∑
cyc
y
√
y√
x(x2 + yz)
≥ 3√
2
Sử dụng bất đẳng thức Cauchy Schwarz, ta có
V T ≥ (x+ y + z)
2√
xy(x2 + yz) +
√
yz(y2 + zx) +
√
zx(z2 + xy)
≥ (x+ y + z)
2√
(xy + yz + zx)(x2 + y2 + z2 + xy + yz + zx)
Mặt khác, theo bất đẳng thức AM - GM thì
8(xy + yz + zx)(x2 + y2 + z2 + xy + yz + zx)
≤ (x2 + y2 + z2 + 3(xy + yz + zx))2 ≤ 16
9
(x+ y + z)2
Từ đây, ta có đpcm. Đẳng thức xảy ra khi và chỉ khi a = b = c.
1.14 [Vasile Cirtoaje] Cho các số dương x, y, z thỏa x2 + y2 + z2 ≥ 3, chứng minh rằng
x5 − x2
x5 + y2 + z2
+
y5 − y2
y5 + z2 + x2
+
z5 − z2
z5 + x2 + y2
≥ 0
Lời giải. Bất đẳng thức đã cho được viết lại như sau∑
cyc
1
x5 + y2 + z2
≤ 3
x2 + y2 + z2
Từ đây, ta suy ra được chỉ cần xét trường hợp x2 + y2 + z2 = 3 là đủ, khi đó, bất đẳng thức tương
đương ∑
cyc
1
x5 − x2 + 3 ≤ 1
Sử dụng bất đẳng thức AM - GM, ta có
x5 =
x6
x
≥ 2x
6
x2 + 1
Đặt a = x2, b = y2, c = z2 thì ta có a+ b+ c = 3 và ta cần chứng minh∑
cyc
1
2a3
a+1 − a+ 3
≤ 1
Hay ∑
cyc
a+ 1
2a3 − a2 + 2a+ 3 ≤ 1
Hay ∑
cyc
(a− 1)2(−2a2 + 3a+ 3)
2a3 − a2 + 2a+ 3 ≥ 0
14 CHƯƠNG 1. CÁC BÀI TOÁN ĐÃ CÓ LỜI GIẢI
Không mất tính tổng quát, giả sử a ≥ b ≥ c, suy ra a ≥ 1 ≥ c. Xét 2 trường hợp
Trường hợp 1. b+ c ≥ 1, suy ra a ≤ 2, khi đó, ta có
−2a2 + 3a+ 3 > 0, −2b2 + 3b+ 3 > 0, −2c2 + 3c+ 3 > 0
Nên kết quả bài toán là hiển nhiên.
Trường hợp 2. b+ c ≤ 1, suy ra a ≥ 2, ta có
(2a3 − a2 + 2a+ 3)− 5(a+ 1) = 2a3 − a2 − 3a− 2 = a3
(
2− 1
a
− 3
a2
− 2
a3
)
≥ a3
(
2− 1
2
− 3
22
− 2
23
)
=
1
2
a3 > 0
Suy ra a+12a3−a2+2a+3 ≤ 15 . Như vậy, ta chỉ cần chứng minh
b+ 1
2b3 − b2 + 2b+ 3 +
c+ 1
2c3 − c2 + 2c+ 3 ≤
4
5
Điều này luôn đúng vì với mọi 1 ≥ x ≥ 0, ta có
x+ 1
2x3 − x2 + 2x+ 3 ≤
2
5
Thật vậy, bất đẳng thức tương đương
4x3 ≥ (x+ 1)(2x− 1)
Nếu x ≤ 12 thì ta có ngay đpcm, nếu x ≥ 12 thì
4x3 − (x+ 1)(2x− 1) ≥ 4x3 − 2(2x− 1) = 2(2x3 − 2x+ 1)
≥ 2(x2 − 2x+ 1) = 2(x− 1)2 ≥ 0
Bất đẳng thức được chứng minh xong. Đẳng thức xảy ra khi và chỉ khi x = y = z = 1.
1.15 Cho n ≥ 3 và a1, a2, ..., an là các số không âm thỏa a21 + a22 + ... + a2n = 1, chứng minh bất đẳng
thức
1√
3
(a1 + a2 + ...+ an) ≥ a1a2 + a2a3 + ...+ ana1
Lời giải. Đặt fn(a1, a2, ..., an) = 1√3 (a1 + a2 + ... + an) − (a1a2 + a2a3 + ... + ana1). Không mất tính
tổng quát, giả sử a1 = max{a1, a2, ..., an}. Nếu an ≤ 1√3 thì
fn(a1, a2, ..., an)− fn−1
(
a1, a2, ..., an−2,
√
a2n−1 + a2n
)
=
1√
3
(
an−1 + an −
√
a2n−1 + a2n
)
+ (an−2 + a1)
√
a2n−1 + a2n − an−1(an−2 + an)− ana1
≥
(
1√
3
− an
)(
an−1 + an −
√
a2n−1 + a2n
)
≥ 0
Suy ra
fn(a1, a2, ..., an) ≥ fn−1
(
a1, a2, ..., an−2,
√
a2n−1 + a2n
)
15
Nếu an ≥ 1√3 thì ta có a1 ≥ 1√3 , suy ra an−1 ≤ 1√3 , do đó
fn(a1, a2, ..., an)− fn−1
(
a1, a2, ..., an−3,
√
a2n−2 + a
2
n−1, an
)
=
1√
3
(
an−2 + an−1 −
√
a2n−2 + a
2
n−1
)
+ an−3
(√
a2n−2 + a
2
n−1 − an−2
)
+ an
(√
a2n−2 + a
2
n−1 − an−1
)
− an−2an−1 ≥ an−2
(
1√
3
− an−1
)
≥ 0
Suy ra
fn(a1, a2, ..., an) ≥ fn−1
(
a1, a2, ..., an−3,
√
a2n−2 + a
2
n−1, an
)
Từ đây, ta suy ra được ta chỉ cần chứng minh bất đẳng thức trong trường hợp n = 3 là đủ nhưng
trong trường hợp này, bất đẳng thức là hiển nhiên nên ta có đpcm. Đẳng thức xảy ra khi và chỉ
khi n = 3 và a1 = a2 = a3 = 1√3 .
1.16 [Nguyễn Anh Cường] Cho các số dương a, b, c, chứng minh bất đẳng thức√
a
b
+
b
c
+
c
a
+
√
ab+ bc+ ca
a2 + b2 + c2
≥
√
3 + 1
Lời giải. Trước hết, ta chứng minh kết quả sau với mọi a, b, c > 0
(a+ b+ c)2
(
a
b
+
b
c
+
c
a
)
≥ 9(a2 + b2 + c2)
Thật vậy, bất đẳng thức tương đương ∑
cyc
Sc(a− b)2 ≥ 0
trong đó
Sa =
b
c
+
a
b
+
2a
c
− 5
2
, Sb =
c
a
+
b
c
+
2b
a
− 5
2
, Sc =
a
b
+
c
a
+
2c
b
− 5
2
Không mất tính tổng quát, giả sử a = max{a, b, c}. Nếu c ≥ b thì ta có ab + bc + ca ≥ ac + cb + ba nên
không mất tính tổng quát, ta chỉ cần xét a ≥ b ≥ c > 0 là đủ, khi đó, dễ thấy Sa ≥ 0. Ta sẽ chứng
minh
Sa + 2Sb ≥ 0, Sc + 2Sb ≥ 0, Sb + Sc ≥ 0
Thật vậy, ta có
Sa + 2Sb = 2
(a
c
+
c
a
)
+
(
a
b
+
4b
a
)
+
3b
c
− 15
2
≥ 4 + 4 + 3− 15
2
> 0
Sc + 2Sb = 2
(
b
c
+
c
b
)
+
(
a
b
+
4b
a
)
+
3c
a
− 15
2
≥ 4 + 4− 15
2
> 0
Sb + Sc =
(
a
b
+
2b
a
)
+
(
b
2c
+
2c
b
)
+
(
b
2c
+
2c
a
)
− 5 ≥ a
b
+ 2
√
b
a
+
2b
a
+ 2− 5
=
(
a
2b
+
2b
a
)
+
(
a
2b
+
√
b
a
+
√
b
a
)
− 3 ≥ 2 + 3
3
√
2
− 3 > 0
Từ đây, ta có
16 CHƯƠNG 1. CÁC BÀI TOÁN ĐÃ CÓ LỜI GIẢI
+, Nếu Sb ≤ 0 thì ∑
cyc
Sc(a− b)2 ≥ (Sa + 2Sb)(b− c)2 + (Sc + 2Sb)(a− b)2 ≥ 0
+, Nếu Sb ≥ 0 thì ∑
cyc
Sc(a− b)2 ≥ (Sc + Sb)(a− b)2 ≥ 0
Bất đẳng thức trên được chứng minh, sử dụng bất đẳng thức này, ta suy ra được, ta chỉ cần chứng
minh
3
√
a2 + b2 + c2
(a+ b+ c)2
+
√
ab+ bc+ ca
a2 + b2 + c2
≥
√
3 + 1
Đặt x =
√
ab+bc+ca
a2+b2+c2 ≤ 1, ta cần chứng minh
3√
2x2 + 1
+ x ≥
√
3 + 1
Dễ dàng kiểm tra được bất đẳng thức này đúng với mọi 1 ≥ x ≥ 0, vậy ta có đpcm. Đẳng thức xảy
ra khi và chỉ khi a = b = c.
1.17 [Nguyễn Văn Thạch] Chứng minh rằng với mọi a, b, c > 0, ta có
a2
b2
+
b2
c2
+
c2
a2
+
8(ab+ bc+ ca)
a2 + b2 + c2
≥ 11
Lời giải. Trước hết, ta sẽ chứng minh kết quả sau với mọi x, y, z > 0 thỏa xyz = 1
x2 + y2 + z2 + 6 ≥ 3
2
(
x+ y + z +
1
x
+
1
y
+
1
z
)
Thật vậy, không mất tính tổng quát, giả sử x = min{x, y, z}. Đặt t = √yz và
P (x, y, z) = x2 + y2 + z2 + 6− 3
2
(
x+ y + z +
1
x
+
1
y
+
1
z
)
Ta có
P (x, y, z)− P (x, t, t) = 1
2
(√
y −√z)2(2 (√y +√z)2 − 3− 3
bc
)
≥ 1
2
(√
y −√z)2 (8− 3− 3) ≥ 0
Lại có
P (x, t, t) = P
(
1
t2
, t, t
)
=
(t− 1)2((t2 − 2t− 1)2 + t2 + 1)
2t4
≥ 0
Bất đẳng thức được chứng minh. Trở lại bài toán của ta, sử dụng bất đẳng thức trên với x = ab , y =
b
c , z =
c
a , ta suy ra được ta chỉ cần chứng minh
3
2
∑
cyc
a2 + b2
ab
+
8(ab+ bc+ ca)
a2 + b2 + c2
≥ 17
Hay ∑
cyc
Sc(a− b)2 ≥ 0
17
trong đó
Sa =
3
bc
− 8
a2 + b2 + c2
, Sb =
3
ca
− 8
a2 + b2 + c2
, Sc =
3
ab
− 8
a2 + b2 + c2
Không mất tính tổng quát, giả sử a ≥ b ≥ c, khi đó, dễ thấy Sa ≥ Sb ≥ Sc, lại có
Sb + Sc =
3(b+ c)
abc
− 16
a2 + b2 + c2
≥ 6
a
√
bc
− 16
a2 + 2bc
≥ 6
a
√
bc
− 16
2a
√
2bc
> 0
Từ đây, ta dễ dàng suy ra đpcm. Đẳng thức xảy ra khi và chỉ khi a = b = c.
Nhận xét. Chú ý rằng
(
ab+bc+ca
a2+b2+c2
)2
+ 1 ≥ 2(ab+bc+ca)
a2+b2+c2
, ta suy ra
a2
b2
+
b2
c2
+
c2
a2
+ 4
(
ab+ bc+ ca
a2 + b2 + c2
)2
≥ 7
Kết quả này được tìm ra bởi bạn Nguyễn Anh Cường và đã được đưa lên 
1.18 [Phạm Thị Hằng] Chứng minh rằng với mọi số dương a1, a2, ..., an, b1, b2, ..., bn, ta có(
n∑
i=1
a2i
)(
n∑
i=1
b2i
)
≥
(
n∑
i=1
bi(ai + bi)
)(
n∑
i=1
a2i bi
ai + bi
)
Lời giải. Đặt fn(a1, a2, ..., an) = V T − V P . Ta sẽ chứng minh bất đẳng thức đã cho bằng quy nạp. Với
n := 1 thì bất đẳng thức là hiển nhiên, giả sử bất đẳng thức đúng với n := n, khi đó, 

File đính kèm:

  • pdftoanhocmuonmau.pdf